Administración     

Olimpiadas de Matemáticas
Página de preparación y problemas

OME Local
OME Andaluza
OME Nacional
OIM
IMO
EGMO
USAMO
ASU
OMCC
Retos UJA
Selector
La base de datos contiene 2434 problemas y 940 soluciones.

XXXIX Olimpiada Iberoamericana de Matemáticas — 2024

Sesión 1 —  21 de septiembre de 2024

Problema 1771
Para cada entero positivo $n$, sea $d(n)$ el número de divisores enteros positivos de $n$. Demostrar que, para todos los pares de enteros positivos $(a,b)$, se tiene que \[d(a)+d(b)\leq d(\mathrm{mcd}(a,b))+d(\mathrm{mcm}(a,b))\] y determinar para qué pares $(a,b)$ se tiene la igualdad.
Sin pistas
Sin soluciones
info
Si crees que el enunciado contiene un error o imprecisión o bien crees que la información sobre la procedencia del problema es incorrecta, puedes notificarlo usando los siguientes botones:
Informar de error en enunciado Informar de procedencia del problema
Problema 1772
Sea $ABC$ un triángulo acutángulo y sean $M$ y $n$ los puntos medios de $AB$ y $AC$, respectivamente. Dado un punto $D$ en el interior del segmento $BC$ tal que $DB\lt DC$, sean $P$ y $Q$ las intersecciones de $DM$ y $DN$ con $AC$ y $AB$, respectivamente. Sea $R\neq A$ el punto de intersección de las circunferencias circunscritas de los triángulos $PAQ$ y $AMN$. Si $K$ es el punto medio de $AR$, demostrar que $\angle MKN=2\angle BAC$.
Sin pistas
Sin soluciones
info
Si crees que el enunciado contiene un error o imprecisión o bien crees que la información sobre la procedencia del problema es incorrecta, puedes notificarlo usando los siguientes botones:
Informar de error en enunciado Informar de procedencia del problema
Problema 1773
Sea $O$ un punto fijo en el plano. Tenemos $2024$ puntos rojos, $2024$ puntos amarillos y $2024$ puntos verdes en el plano, no habiendo tres de ellos alineados y siendo todos distintos de $O$. Se sabe que para cualesquiera dos colores, la envolvente convexa de los puntos que son de alguno de estos dos colores contiene a $O$ (en el interior o en el borde). Diremos que un punto rojo, un punto amarillo y un punto verde forman un triángulo boliviano si dicho triángulo contiene a $O$ en el interior o en el borde. Hallar el mayor entero positivo $k$ tal que, independientemente de como estén colocados los puntos, siempre hay al menos $k$ triángulos bolivianos.
Sin pistas
Sin soluciones
info
Si crees que el enunciado contiene un error o imprecisión o bien crees que la información sobre la procedencia del problema es incorrecta, puedes notificarlo usando los siguientes botones:
Informar de error en enunciado Informar de procedencia del problema

Sesión 2 —  22 de septiembre de 2024

Problema 1774
Coloreamos algunos puntos del plano de rojo de forma que si $P$ y $Q$ están coloreados y $X$ es un punto tal que el triángulo $PQX$ tiene ángulos de $30^\circ,60^\circ,90^\circ$ (en algún orden), entonces $X$ también está coloreado. Si tenemos tres puntos $A,B,C$ coloreados, demostrar que el baricentro del triángulo $ABC$ también está coloreado.
Sin pistas
Sin soluciones
info
Si crees que el enunciado contiene un error o imprecisión o bien crees que la información sobre la procedencia del problema es incorrecta, puedes notificarlo usando los siguientes botones:
Informar de error en enunciado Informar de procedencia del problema
Problema 1775
Sea $n\geq 2$ un entero y sean $a_1,a_2,\ldots,a_n$ enteros positivos fijos (no necesariamente distintos) de forma que $\mathrm{mcd}(a_1,a_2,\ldots,a_n)=1$. En una pizarra se escriben todos los números junto con un entero positivo $x$. Un movimiento consiste en elegir dos números $a\gt b$ de los $n+1$ números de la pizarra y reemplazarlos con $a-b$ y $2b$. Encontrar todos los posibles valores de $x$ (en función de $a_1,a_2,\ldots,a_n$) para los que es posible hacer que todos los números escritos en la pizarra sean iguales después de un número finito de movimientos.
Sin pistas
Sin soluciones
info
Si crees que el enunciado contiene un error o imprecisión o bien crees que la información sobre la procedencia del problema es incorrecta, puedes notificarlo usando los siguientes botones:
Informar de error en enunciado Informar de procedencia del problema
Problema 1776
Hallar todos los conjuntos infinitos $A$ de enteros positivos que cumplen que \[\Bigl\lfloor\frac{a}{b}\Bigr\rfloor\in A\text{ para cualesquiera } a,b\in A\text{ con }a\geq b.\]
Sin pistas
Sin soluciones
info
Si crees que el enunciado contiene un error o imprecisión o bien crees que la información sobre la procedencia del problema es incorrecta, puedes notificarlo usando los siguientes botones:
Informar de error en enunciado Informar de procedencia del problema
José Miguel Manzano © 2010-2025. Esta página ha sido creada mediante software libre